Question 1 (I dont remember the exact question)
Let \( f(x) = sin(x) \) and \( g(x) = cos(x) \). What's the equivalent of \( f(x) = \frac{\frac{f(x) + g(x)}{f(x) - g(x)} + \frac{f(x) - g(x)}{f(x) + g(x)}}{\frac{f(x) + g(x)}{f(x) - g(x)} - \frac{f(x) - g(x)}{f(x) + g(x)}} \)?
Answer: c) \( \frac{1}{\sin(2\theta)} \)
Question 2
In the point x=\(\frac{\pi}{4}\), the function from 1:
- a) has an infinite discontinuity
- b) has a jump
- c) has a removable discontinuity
- d) is continuous
At first glance I was mistaken by \( sin(\frac{\pi}{2}) = 1 \) - I assumed the function is continuous and \( lim_{x \to \frac{\pi}{4}} \frac{1}{sin(2x)} = \frac{1}{sin(\frac{\pi}{2})} = 1 \)
However, if you place \( x = \frac{\pi}{4} \) in the original function, you get \( f(x) = \frac{\frac{f(x) + g(x)}{\underbrace{f(x) - g(x)}}_{=0} + \frac{f(x) - g(x)}{f(x) + g(x)}}{\frac{f(x) + g(x)}{\underbrace{f(x) - g(x)}_{=0}} - \frac{f(x) - g(x)}{f(x) + g(x)}} \), since at \( x = \frac{\pi}{4} \), we have: \[ f(x) - g(x) = sin(\frac{\pi}{4}) - cos(\frac{\pi}{4}) = \frac{1}{\sqrt{2}} - \frac{1}{\sqrt{2}} = 0 \] So the function is not continuous at \( x = \frac{\pi}{4} \)
Answer: c) has a removable discontinuity
Question 3
Calculate:
\[ \lim_{x \to \frac{1}{4}} \frac{\frac{1}{x} - 4}{x - \frac{1}{4}} \]
Answer: b) -16
Explanation:
- For simplicity let \( t = \frac{1}{x} \), so the limit becomes: \[ \lim_{t \to 4} \frac{t - 4}{\frac{1}{t} - \frac{1}{4}} \]
- We can simplify the expression: \[ \lim_{t \to 4} \frac{t - 4}{\frac{4 - t}{4t}} \]
- So the limit becomes: \[ \lim_{t \to 4} \frac{4t \cdot (t - 4)}{(-1) \cdot (t - 4)} = \lim_{t \to 4} -4t = -16 \text{ (since } t = \frac{1}{x} \text{)} \]
We can also use the sandwich theorem: \[ \frac{-4}{x} \leq \frac{\frac{1}{x} - 4}{x - \frac{1}{4}} \leq \frac{\frac{1}{x}}{- \frac{1}{4}} \] Since \( \lim_{x \to \frac{1}{4}} \frac{-4}{x} = -16 \) and \( \lim_{x \to \frac{1}{4}} \frac{\frac{1}{x}}{- \frac{1}{4}} = -16 \), the limit is -16 as well.
Question 4 (Not sure about that one)
Let \( f(x) \). be:
\[ f(x) = \begin{cases} \frac{\sin(2x)}{x^3 + 2x^2 - 3x} & \text{if } x \neq 0 \\ - \frac{2}{3} & \text{if } x = 0 \end{cases} \] In what interval the function get total minimum and maximum values?
Answer: a) \((-3, 1)\)
Explanation:
We can simplify the function to:
\[ f(x) = \begin{cases} \frac{\sin(2x)}{x(x^2 + 2x - 3)} & \text{if } x \neq 0 \\ - \frac{2}{3} & \text{if } x = 0 \end{cases} \]
I dont rember the question well but I did notice that:
- The function is continuous in the open interval (-3,1), but not in the closed interval [-3,1]
- The function is not defined at x = -3, 1
- The function is continuous at x = 0
Question 5
Calculate: Whats the inverse function of \( f(x) = 2^x + 2^{-x} \)?
Answer: d) \( f^{-1}(x) = \log_2(x + \sqrt{x^2 - 4}) - 1 \)
We can use \( sinh(x) = \frac{e^x - e^{-x}}{2} \) or just try to solve it by ourselves
Question 6
The domain fo \( f^{-1}(x) \) for the prev question is:
Answer: d) \( x \geq 2 \)
No matter what, the function is defined for \( x \geq 2 \)
Question 7
The solutions to the equation \( f(x) - 4 = 0 \) are:
Answer: c) \( x = log_2(2 + - \sqrt{3}) \)
Placing \(x = log_2(2 + - \sqrt{3})\) in the function, we get \(2^{log_2(2 + - \sqrt{3})} + 2^{-log_2(2 + - \sqrt{3})} = 4\)
Question 8
Whats the domain fo \( f(x) = \tan(tanh(x)) \)?
Answer: b) \( R \)
Since the tahnh function is defined for all real numbers, and maps them to (-1, 1), no problem with the tan function (wich is defined for evenry x except \( \frac{\pi}{2} + \pi n \)), the domain is \( R \)
Question 9
For \( f(x) \) from the previous question, wich is true?
Answer: a) \( tanh^{-1}(tan^{-1}(sin(x))) = sin(x) \)
Placing \( sin(x) \) in the function, we get \( tanh^{-1}(tan^{-1}(tan(tanh(sin(x))))) = sin(x) \)
Question 10
Calculate: \( \lim_{x \to \pi / 2} \sin(x) ^ \frac{\pi}{cos^2(x)} \)
Answer: c) \( \lim_{x \to \pi / 2} \sin(x) ^ \frac{\pi}{cos^2(x)} = e^{-\pi/2} \)
Explanation:
The expression is similar to \( 1^\infty \), so we can use the limit \( \lim_{x \to 0} (1 + x)^\frac{1}{x} = e \)
To do so, notice that \( \sin x = \left( 1 - {\cos^2 x} \right) ^ {1/2} \) since \( \sin^2 x = 1 - \cos^2 x \)
So the limit becomes: \[ \lim_{x \to \pi / 2} \left[\left( 1 - {\cos^2 x} \right) ^{\frac{1}{2}} \right]^{\frac{\pi}{\cos^2 x}} \]
Rearranging the expression, we get: \[ \lim_{x \to \pi / 2} \left[\left( 1 - {\cos^2 x} \right) ^{\frac{1}{ \cos^2 x}}\right]^{\frac{\pi}{2}} \]
We can take out \( \frac{\pi}{2} \) from the limit, since a constant factor doesn't affect the limit, and we get: \[ \left[ \lim_{x \to \pi / 2} \left( 1 - {\cos^2 x} \right) ^{\frac{1}{ \cos^2 x}} \right]^{\frac{\pi}{2}} \]
We can replace \( -\cos^2 x \) with \( y = -\cos^2 x \), so the limit becomes: \[ \left[ \lim_{y \to 0} \left( 1 + y \right) ^{\frac{1}{y}} \right]^{-\left(\frac{\pi}{2}\right)} = \boxed{e^{-\pi/2}} \]
Alternative Solution (L'Hopital's Rule)
We can declare: \[ y = \left(\sin x\right)^{\frac{\pi}{\cos^2 x}} \Rightarrow \ln y = \frac{\pi}{\cos^2 x} \ln \left( \sin x\right)\] And we get: \[ \lim_{x \to \pi / 2} \left( \ln y \right) = \pi \lim_{x \to \pi / 2} \left( \frac{\ln \left( \sin x\right)}{\cos^2 x}\right) \]
This expression is from the form \( \frac{0}{0} \), so we can use L'Hopital's Rule: \[ \pi \lim_{x \to \pi / 2} \left( \frac{\ln \left( \sin x\right)}{\cos^2 x}\right) = \pi \lim_{x \to \pi / 2} \left( \frac{\frac{ \cancel{ \cos x }}{\sin x}}{-2 \cancel{ \cos x} \sin x}\right) = \pi \lim_{x \to \pi / 2} \left( \frac{-1}{2 \sin^2 x}\right) = - \frac{\pi}{2} \]
So we get: \[ \ln y = - \frac{\pi}{2} \Rightarrow y = e^{-\pi/2} \]
Question 11
Given continuous functions f, g at point x₀, and function h with limit L at x₀, find: \[\lim_{x \to x_0} f\left(g^{h\left(x\right)}\right)\]
- Let's analyze this step by step. We have: - f, g are continuous at point x₀ - h has limit L at x₀
- Looking at the innermost function: \[\lim_{x \to x_0} h(x) = L\]
- Since g is continuous at x₀, then: \[\lim_{x \to x_0} g^{h(x)} = g^{\lim_{x \to x_0} h(x)} = g^L\]
- Finally, since f is continuous, we can apply it: \[\lim_{x \to x_0} f\left(g^{h(x)}\right) = f\left(\lim_{x \to x_0} g^{h\left(x\right)}\right) = f(g^L)\]
Answer: a) f(g^L)
Summery
- Question 1: c) \( \frac{1}{\sin(2\theta)} \)
- Question 2: c) has removable point
- Question 3: b) -16
- Question 4: a) \((-3, 1)\) (not sure about that one)
- Question 5: d) \( f^{-1}(x) = \log_2(x + \sqrt{x^2 - 4}) - 1 \)
- Question 6: d) \( x \geq 2 \)
- Question 7: c) \( x = log_2(2 + - \sqrt{3}) \)
- Question 8: b) \( R \)
- Question 9: a) \( tanh^{-1}(tan^{-1}(sin(x))) = sin(x) \)
- Question 10: ?) \( e^{-\pi/2} \) (not sure and dont remember the exact solution)
- Question 11: a) f(g^L)